《数学分析》笔记二(极限)

\(\newcommand{\bbR}{\mathbb{R}}\) \(\newcommand{\bbN}{\mathbb{N}^+}\) \(\newcommand{\Lim}{\lim\limits_}\) \(\newcommand{\Sum}{\sum\limits_}\) \(\newcommand{\eps}{\varepsilon}\) \(\newcommand{\Colon}{\colon\;\;}\)

以下数列仅讨论实数列。

一些理论

数列收敛性

数列 \(\{a_n\}\) 收敛于 \(A\) ,当且仅当 \(\forall \eps > 0 \;\; \exists N \in \bbN \;\; \forall n > N \;\; |a_n - A| < \eps\)

一个显然等价的定义是对于 \(A\) 的任何领域,数列中仅有有限项的值不被该领域包含。注意到该定义并不关心数列中的数的排列顺序,因此把数列按任意次序打乱不影响其敛散性。


数列 \(\{a_n\}\) 是柯西数列,当且仅当 \(\forall \eps > 0 \;\; \exists N \in \bbN \;\; \forall n, m > N \;\; |a_n - a_m| < \eps\)

数列收敛当且仅当其是柯西数列,由于柯西数列的定义并不关心数列收敛于何值,常用于判定数列是否收敛。


单调数列收敛当且仅当其有界。

将数列去掉任意有限项后其收敛性以及其收敛的值都不会改变。因此若数列在某一确定项之后开始单调(即“最终单调”),该数列收敛当且仅当其有界。

而无界单调数列一定趋于无穷。


两个收敛数列逐项相加/减/乘/除,新数列收敛于原来两个数列极限的和/差/积/商。

收敛子列

由聚点定理可以知道:任意有界数列存在收敛子列。

另外,任意无界数列存在趋于无穷的子列。

而数列收敛于 \(x\) 当且仅当其任何子列都收敛于 \(x\)


\(x\) 是数列 \(\{a_n\}\) 的部分极限,当且仅当存在该数列的某个子列收敛于 \(x\)

数列 \(a\) 的下极限 \(\varliminf a_n = \Lim{N \to \infty} \inf \{a_n | n > N\}\)\(a\) 的所有部分极限的最小值。

数列 \(a\) 的上极限 \(\varlimsup a_n = \Lim{N \to \infty} \sup \{a_n | n > N\}\)\(a\) 的所有部分极限的最大值。

那么数列收敛当且仅当上下极限相等。

如果把趋于正负无穷的数列看作是收敛于 \(+\infty\)\(-\infty\) ,上述命题仍是成立的。

无穷级数

数列 \(\{a_n\}\) 的无穷级数 \(\Sum{n=1}^{\infty} a_n := \Lim{n \to \infty} \Sum{k=1}^n a_k\) ,若后者发散,则该级数发散。

级数收敛的必要条件是 \(\Lim{n \to \infty} a_n = 0\)

如果总有 \(|a_n| \le b_n\)\(\Sum{n=1}^{\infty} b_n\) 收敛,那么 \(\Sum{n=1}^{\infty} |a_n|\) 一定收敛。

\(\Sum{n=1}^{\infty} |a_n|\) 收敛蕴含了 \(\Sum{n=1}^{\infty} a_n\) 收敛。


对于无穷级数 \(\Sum{n=1}^{\infty} a_n\) ,令 \(A := \varlimsup \sqrt[n]{|a_n|}\) (或 \(A := \Lim{n \to \infty} |\dfrac{a_{n+1}}{a_n}|\) ,如果该极限存在):

  • \(A < 1 \Rightarrow \Sum{n=1}^{\infty} a_n\) 绝对收敛。
  • \(A > 1 \Rightarrow \Sum{n=1}^{\infty} a_n\) 发散。

一些习题

三.1.4.e

证明无穷连分数 \([a_1, a_2, a_3 \dots]\) \((a_i \in \bbN)\) 总是收敛的。

ps: 渐进分数 \([a_1, a_2 \dots a_n]\) 的值可以表示为 \(\dfrac{P_n}{Q_n}\) ,并满足 \(P_{n+2} = P_{n+1} a_{n+2} + P_n\) 以及 \(Q_{n+2} = Q_{n+1} a_{n+2} + Q_n\)

Answer

\(R_n := \dfrac{P_n}{Q_n}\) ,我们要证明 \(\Lim{n \to \infty} R_n\) 存在。

首先注意到

\[ \begin{aligned} P_{n+2} Q_{n+1} - P_{n+1} Q_{n+2} &= (P_{n+1} a_{n+2} + P_n) Q_{n+1} - P_{n+1} (Q_{n+1} a_{n+2} + Q_n) \\ &= P_n Q_{n+1} - P_{n+1} Q_n \end{aligned} \]

迭代上式后可以得到 \(P_{n+1} Q_n - P_n Q_{n+1} = (-1)^{n-1} (P_2 Q_1 - P_1 Q_2) = (-1)^{n-1}\)

那么对相邻项作差:

\[ R_{n+1} - R_n = \frac{P_{n+1}}{Q_{n+1}} - \frac{P_n}{Q_n} = \frac{(-1)^{n+1}}{Q_{n+1} Q_n} \]

这启发我们按将下标按奇偶性分类,于是

\[ \begin{aligned} R_{2k+1} - R_{2k-1} &= \frac{(-1)^{2k+1}}{Q_{2k+1} Q_{2k}} + \frac{(-1)^{2k}}{Q_{2k} Q_{2k-1}} \\ &= \frac{a_{2k+1}}{Q_{2k+1} Q_{2k-1}} \\ &> 0 \\ R_{2k+2} - R_{2k} &= \frac{(-1)^{2k+2}}{Q_{2k+2} Q_{2k+1}} + \frac{(-1)^{2k+1}}{Q_{2k+1} Q_{2k}} \\ &= - \frac{a_{2k+2}}{Q_{2k+2} Q_{2k}} \\ &< 0 \end{aligned} \]

\(R_{2k-1}\) 单调递增,\(R_{2k}\) 单调递减。

且由于 \(R_{2k} - R_{2k-1} > 0\) ,所以 \(R_{2k-1} < R_{2k} \le R_2\)\(R_2\)\(R_{2k-1}\) 的一个上界。同理 \(R_1\)\(R_{2k}\) 的一个下界。故 \(\Lim{k \to \infty} R_{2k}\)\(\Lim{k \to \infty} R_{2k-1}\) 都是存在的。于是

\[ \Lim{k \to \infty} R_{2k} - \Lim{k \to \infty} R_{2k-1} = \Lim{k \to \infty} (R_{2k} - R_{2k-1}) = \Lim{k \to \infty} \frac{(-1)^{2k}}{Q_{2k} Q_{2k-1}} = 0 \]

所以奇数项和偶数项收敛于同值 \(r\) ,这就可以导出 \(R_n\) 收敛于 \(r\)

\[ \begin{aligned} & \because \forall \eps > 0 \;\; \exists N, M \;\; \forall n > N, m > M \;\; \max\{|R_{2n-1} - r|, |R_{2m} - r|\} < \eps \\ & \therefore \forall \eps > 0 \;\; \exists K = \max\{2N-1, 2M\} \;\; \forall n > K \;\; |R_n - r| < \eps \end{aligned} \]

三.1.6.b

对于 \(0 < a \le b\)\(\Lim{n \to \infty} \sqrt[n]{\dfrac{a^n + b^n}{2}}\)

Answer

\(r_n := \sqrt[n]{\dfrac{a^n + b^n}{2}}\) ,注意到 \(\dfrac{r_n}{b} = \sqrt[n]{\dfrac{(a/b)^n + 1}{2}}\) ,令 \(q := \dfrac{a}{b}\) ,则 \(0 < q \le 1\)

注意到 \(1 < q^n + 1 \le 2\) 可以导出以下不等式:

\[ \sqrt[n]{\dfrac{1}{2}} < \sqrt[n]{\dfrac{q^n + 1}{2}} \le \sqrt[n]{\dfrac{2}{2}} \]

而左式和右式的极限都是 \(1\) ,故 \(\Lim{n \to \infty} \dfrac{r_n}{b} = 1\) ,于是 \(\Lim{n \to \infty} r_n = b\)

三.1.7

数列 \(\{x_n\}\) 满足 \(x_{n+1} = \dfrac{x_n^2 + a}{2 x_n}\)\(x_1, a > 0\) ,求 \(\Lim{n \to \infty} x_n\)

Answer

注意到 \(x_{n+1} = \dfrac{1}{2} (\sqrt{x_n} - \sqrt{\dfrac{a}{x_n}})^2 + \sqrt{a} \ge \sqrt{a}\) ,不失一般性地,不妨假定 \(x_1 \ge \sqrt{a}\)

\(x_1 = \sqrt{a}\) 时显然 \(\Lim{n \to \infty} x_n = \sqrt{a}\) ,考虑 \(x_1 > \sqrt{a}\)

\(\Delta_n := x_n - \sqrt{a}\) ,那么

\[ \Delta_{n+1} = \frac{x_n^2 + a}{2 x_n} - \sqrt{a} = \frac{\Delta_n^2 + 2 \sqrt{a} \Delta_n + 2 a}{2 \Delta_n + 2 \sqrt{a}} - \sqrt{a} = \frac{\Delta_n^2}{2 \Delta_n + 2 \sqrt{a}} \]

可以发现 \(\Delta_n > 0\) 恒成立,于是

\[ \Delta_{n+1}^{-1} = \frac{2 \Delta_n + 2 \sqrt{a}}{\Delta_n^2} = 2 \Delta_n^{-1} + 2 \sqrt{a} \Delta_n^{-2} > 2 \Delta_n^{-1} > 2^n \Delta_1^{-1} \]

于是有不等式 \(0 < \Delta_{n+1} < \dfrac{\Delta_1}{2^n}\) ,左式和右式的极限均为 \(0\) ,故 \(\Lim{n \to \infty} \Delta_n = 0\) ,即 \(\Lim{n \to \infty} x_n = \sqrt{a}\)

另外,我们顺便求得了 \(\{x_n\}\) 的收敛速度,而这个逼近的方法正是牛顿迭代。

补充:狄利克雷检验法

已知单调数列 \(\{a_n\}\) 收敛于 \(0\) 且级数 \(\Sum{n=1}^{\infty} b_n\) 的部分和序列 \(\{\Sum{k=1}^n b_k\}\) 有界,证明级数 \(\Sum{n=1}^{\infty} a_n b_n\) 收敛。

Answer

(其实是抄的 Wikipedia 的证明)

\(S_n := \Sum{k=1}^n a_k b_k\) 以及 \(B_n := \Sum{k=1}^n b_k\)

注意到

\[ S_n = \sum_{k=1}^n a_k (B_k - B_{k-1}) = \sum_{k=1}^n a_k B_k - \sum_{k=1}^{n-1} a_{k+1} B_k = a_n B_n + \sum_{k=1}^{n-1} (a_k - a_{k+1}) B_k \]

\(\forall n \in \bbN \Colon |B_n| < L\) ,注意到

\[ \sum_{k=1}^n |(a_k - a_{k+1}) L| = L |\sum_{k=1}^n (a_k - a_{k+1})| = L |a_1 - a_{n+1}| \]

故级数 \(\Sum{n=1}^{\infty} |(a_n - a_{n+1}) L|\) 收敛于 \(|L a_1|\) 。而根据比较定理,级数 \(\Sum{n=1}^{\infty} |(a_n - a_{n+1}) B_n|\) 也一定收敛。

那么级数 \(\Sum{n=1}^{\infty} (a_n - a_{n+1}) B_n\) 收敛,又因为 \(a_n B_n\) 也收敛于 \(0\) ,故 \(S_n\) 收敛,即原级数收敛。

二些理论

函数的极限

对于函数 \(f \colon E \to \bbR\) ,称在 \(x\) 趋于 \(a\) 时函数 \(f\) 趋于 \(A\) ,当且仅当 \(\forall U(A) \;\; \exists \mathring{V}(a) \;\; f(\mathring{V}(a) \cap E) \subset U(A)\) 。记为 \(\Lim{E \ni x \to a} f(x) = A\) 。如果存在 \(\mathring{V}(a) \subset E\) ,一般简记为 \(\Lim{x \to a} f(x) = A\)

注意到满足上述定义的一个必要条件是 \(a\)\(E\) 的一个极限点。

另一个等价的定义是,所有满足 \(x_n \in E \setminus \{a\}\) 且收敛于 \(a\) 的数列 \(\{x_n\}\) 都有 \(\{f(x_n)\}\) 收敛于 \(A\)

根据第二个定义可以很快地将数列极限的部分理论推广到函数极限上(接下来将会看到)。

滤子基

由集合 \(E\) 的某些子集 \(B \subset E\) 组成的集合族 \(\mathcal B\) 称为 \(E\) 的滤子基(下文简称为基),当且仅当:

  1. \(\forall B \in \mathcal B \;\; B \neq \varnothing\)
  2. \(\forall B_1, B_2 \in \mathcal B \;\; \exists B \in \mathcal B \;\; B \subset B_1 \cap B_2\)

事实上可以直接在基上定义极限:

\[ (\Lim{\mathcal B} f(x) = A) := \forall U(A) \;\; \exists B \in \mathcal B \Colon f(B) \subset U(A) \]

而数列极限和函数极限都可以归约到上述定义。例如对于数列极限 \(\Lim{n \to \infty} a_n = A\)\(\{n \in \bbN | n > N\} \in \mathcal B, f(n) = a_n\) 。函数极限 \(\Lim{E \ni x \to a} f(x) = A\)\(\mathring{V}(a) \cap E \in \mathcal B\)


一些数列极限的相关理论现在可以推广。

  • 对于基 \(\mathcal B\)\(f(x)\) 收敛当且仅当 \(\forall \eps > 0 \;\; \exists B \in \mathcal B \;\; \forall x_1, x_2 \in B \Colon |f(x_1) - f(x_2)| < \eps\)

  • 不减函数 \(f \colon E \to \bbR\)\(E \ni x \to \sup E\) 收敛当且仅当其值有上界(这里的基不是任意的!)。

  • 对于一个确定的基,两个函数相加/减/乘/除,新的函数收敛于原来两个函数的极限的和/差/积/商。


如果某个条件在基 \(\mathcal B\) 上的某个元素 \(B\) 上成立,就称该条件在 \(\mathcal B\) 上最终成立。

对于基 \(\mathcal B\) :记 \(f = o(g)\) 当且仅当存在 \(\Lim{\mathcal B} \alpha(x) = 0\) 使得 \(f(x) = \alpha(x) g(x)\) 最终成立;记 \(f = O(g)\) 当且仅当存在最终有界函数 \(\beta(x)\) 使得 \(f(x) = \beta(x) g(x)\) 最终成立;记 \(f \sim g\) 当且仅当存在 \(\Lim{\mathcal B} \gamma(x) = 1\) 使得 \(f(x) = \gamma(x) g(x)\) 最终成立。

复合函数极限定理

\(\Lim{\mathcal B_Y} g(y) = \Lim{\mathcal B_X} g(f(x))\) ,如果 \(\forall B_Y \in \mathcal B_Y \;\; \exists B_X \in \mathcal B_X \;\; f(B_X) \subset B_Y\)

即如果 \(\Lim{E \ni x \to a} f(x) = b\)\(\Lim{y \to b} g(y) = \Lim{E \ni x \to a} g(f(x))\)

一些需要牢记的公式

\[ \begin{aligned} e^x &= \sum_{n=0}^{\infty} \frac{1}{n!} x^n, & x \in \bbR \\ \cos x &= \sum_{n=0}^{\infty} \frac{(-1)^n}{(2n)!} x^{2n}, & x \in \bbR \\ \sin x &= \sum_{n=0}^{\infty} \frac{(-1)^n}{(2n+1)!} x^{2n+1}, & x \in \bbR \\ \ln (1 + x) &= \sum_{n=0}^{\infty} \frac{(-1)^n}{n+1} x^{n+1}, & |x| < 1 \\ (1 + x)^{\alpha} &= \sum_{n=0}^{\infty} \frac{\alpha^{\underline n}}{n!} x^n, & |x| < 1 \\ \end{aligned} \]

二些习题

三.2.4

证明在 \(\bbN \ni n \to \infty\) 上有 \(\Sum{k=1}^n \dfrac{1}{k} = \ln n + c + o(1)\) ,其中 \(c \in \bbR\) 是某常数。

Answer

\(a_n := \Sum{k=1}^n \dfrac{1}{k} - \ln n\) ,命题等价于数列 \(\{a_n\}\) 收敛。

\(b_n := a_{n+1} - a_n\) ,则

\[ \begin{aligned} b_n &= \frac{1}{n+1} - \ln \frac{n+1}{n} \\ &= \frac{1}{n+1} - \ln (1 + \frac{1}{n}) \\ &= \frac{1}{n+1} - \frac{1}{n} + O(\frac{1}{n^2}) \\ &= - \frac{1}{n(n+1)} + O(\frac{1}{n^2}) \\ &= O(\frac{1}{n^2}) \\ &= \beta_n \frac{1}{n^2} \end{aligned} \]

其中 \(\{\beta_n\}\) 是有界数列。那么

\[ \Lim{n \to \infty} a_n = a_1 + \sum_{n=1}^{\infty} b_n = a_1 + \sum_{n=1}^{\infty} \beta_n \frac{1}{n^2} \]

所以命题等价于级数 \(\Sum{n=1}^{\infty} \dfrac{\beta_n}{n^2}\) 收敛。

\(|\beta_n| < B\) 恒成立,根据比较定理,由于总有 \(|\dfrac{\beta_n}{n^2}| < \dfrac{B}{n^2}\) 且后者的无穷级数收敛,那么原级数绝对收敛。

三.2.5.a

如果两个级数 \(\Sum{n=1}^{\infty} a_n\), \(\Sum{n=1}^{\infty} b_n\) 都是 正项级数,并且当 \(n \to \infty\)\(a_n \sim b_n\) ,则这两个级数同时敛散。

Answer

如果 \(a_n \sim b_n\) 则最终有 \(a_n \le 2 b_n\)\(b_n \le 2 a_n\) 成立,由于它们都是正项级数,根据比较定理可以立刻得出这两个级数同时敛散。


补充) 构造两个数列 \(\{a_n\}\), \(\{b_n\}\) 使得 \(n \to \infty\)\(a_n \sim b_n\) 且它们的无穷级数敛散性不同。

Answer

\(\{b_n\}\) 为数列 \(1, -1, \sqrt{\frac{1}{2}}, -\sqrt{\frac{1}{2}}, \sqrt{\frac{1}{3}}, -\sqrt{\frac{1}{3}}, \sqrt{\frac{1}{4}}, -\sqrt{\frac{1}{4}} \ldots\)

不难发现 \(\Sum{n=1}^{\infty} b_n = 0\) 。并且 \(\Lim{n \to \infty} b_n = 0\)

然后令 \(a_n := b_n + b_n^2\) ,那么

\[ \lim_{n \to \infty} \frac{a_n}{b_n} = \lim_{n \to \infty} (1 + b_n) = 1 \]

\(a_n \sim b_n\)

\(\{b_n\}\) 的无穷级数收敛但 \(\{b_n^2\}\) 的无穷级数发散(调和级数),故它们的和数列的无穷级数 \(\Sum{n=1}^{\infty} a_n\) 发散。

这个例子说明了原题中“正项级数”这个条件是不可忽略的。

三.2.5.b

证明级数 \(\Sum{n=1}^{\infty} \sin \dfrac{1}{n^p}\) 仅当 \(p > 1\) 时收敛。

Answer

\(p > 0\) 时:

\[ \Lim{n \to \infty} n^p \sin \frac{1}{n^p} = \Lim{x \to 0} \frac{\sin x}{x} = 1 \Rightarrow \sin \frac{1}{n^p} \sim \frac{1}{n^p} \]

此时原级数收敛仅当级数 \(\Sum{n=1}^{\infty} \dfrac{1}{n^p}\) 收敛,仅当 \(p > 1\)

\(p = 0\) 时级数 \(\Sum{n=1}^{\infty} \sin 1\) 显然不收敛。

\(p < 0\) 时数列 \(\{\sin n^{-p}\}\) 发散(具体证明?),则级数必然不收敛。

三.2.6.a

如果 \(\forall n \in \bbN \Colon a_n \ge a_{n+1} > 0\) 且级数 \(\Sum{n=1}^{\infty} a_n\) 收敛,证明 \(n \to \infty\)\(a_n = o(\dfrac{1}{n})\)

Answer

命题等价于 \(\Lim{n \to \infty} n a_n = 0\)

\(s_n := \Sum{k=1}^n a_k\) ,由于数列 \(\{s_n\}\) 收敛,那么它是一个柯西数列,于是

\[ \forall \eps > 0 \;\; \exists N \in \bbN \;\; \forall n > N \Colon s_{2n} - s_n < \frac{\eps}{2} \]

而注意到 \(s_{2n} - s_n = a_{n+1} + a_{n+2} + \cdots + a_{2n} \ge n a_{2n}\) ,于是

\[ \forall \eps > 0 \;\; \exists N_1 \in \bbN \;\; \forall n > N_1 \Colon 2 n a_{2n} < \eps \]

同理由 \(s_{2n+1} - s_n \ge (n + 1) a_{2n+1} > (n + \dfrac{1}{2}) a_{2n+1}\) 可以知道

\[ \forall \eps > 0 \;\; \exists N_2 \in \bbN \;\; \forall n > N_2 \Colon (2n + 1) a_{2n+1} < \eps \]

整理可得

\[ \forall \eps > 0 \;\; \exists N = \max\{2 N_1, 2 N_2 + 1\} \;\; \forall n > N \Colon n a_n < \eps \]

三.2.6.b

如果 \(b_n = o(\dfrac{1}{n})\) ,则总是可以构造出一个收敛级数 \(\Sum{n=1}^{\infty} a_n\) ,使得当 \(n \to \infty\)\(b_n = o(a_n)\)

Answer

(不少错误的证法在 \(b_n = \dfrac{1}{n \ln n}\) 时都很容易发现问题,因为级数 \(\Sum{n=1}^{\infty} \dfrac{1}{n \ln n}\) 发散)

(我太菜了,看了 stackexchange 才懂)

\(B_n := \sup\limits_{k \ge n} |k b_k|\) ,则数列 \(\{B_n\}\) 单调不增,且 \(\Lim{n \to \infty} B_n = \varlimsup |n b_n| = \Lim{n \to \infty} |n b_n| = 0\)

\(a_n := (-1)^n B_n\) ,根据狄利克雷检验法可以知道其对应的无穷级数确实收敛,现在我们证明 \(b_n = o(a_n)\)

\[ |a_n| = B_n \ge |n b_n| = n |b_n| \]

若数列 \(\{b_n\}\) 有无限项非零,则 \(- \dfrac{1}{n} \le \dfrac{b_n}{a_n} \le \dfrac{1}{n}\) 于是 \(\Lim{n \to \infty} \dfrac{b_n}{a_n} = 0\) ,则 \(b_n = o(a_n)\)

若数列 \(\{b_n\}\) 仅有有限项非零,则 \(a_n = b_n = 0\) 最终成立,则 \(b_n = o(a_n)\)

三.2.7.b

级数 \(\Sum{n=1}^{\infty} \ln (1 + a_n)\) (其中 \(|a_n| < 1\)) 绝对收敛的充要条件是级数 \(\Sum{n=1}^{\infty} a_n\) 绝对收敛。

Answer

注意到 \(x \to 0\) 时有 \(\ln (1 + x) \sim x\)\(e^x - 1 \sim x\)

\[ \lim_{x \to 0} \frac{\ln (1 + x)}{x} = \lim_{x \to 0} \frac{x + o(x)}{x} = \lim_{x \to 0} (1 + o(1)) = 1 \]

\[ \lim_{x \to 0} \frac{e^x - 1}{x} = \lim_{x \to 0} \frac{1 + x + o(x) - 1}{x} = \lim_{x \to 0} (1 + o(1)) = 1 \]

若第一个级数绝对收敛,则 \(\Lim{n \to \infty} \ln(1 + a_n) = 0\) ,由复合函数极限定理可以知道

\[ \lim_{n \to \infty} \left| \frac{\ln(1 + a_n)}{a_n} \right| = \lim_{n \to \infty} \frac{\ln(1 + a_n)}{a_n} = \lim_{n \to \infty} \frac{\ln(1 + a_n)}{e^{\ln(1 + a_n)} - 1} = \lim_{x \to 0} \frac{x}{e^x - 1} = 1 \]

\(|\ln(1 + a_n)| \sim |a_n|\) ,于是第二个级数也绝对收敛。

同理,若第二个级数绝对收敛,则 \(\Lim{n \to \infty} a_n = 0\) ,由复合函数极限定理可以知道

\[ \lim_{n \to \infty} \left| \frac{\ln(1 + a_n)}{a_n} \right| = \lim_{n \to \infty} \frac{\ln(1 + a_n)}{a_n} = \lim_{x \to 0} \frac{\ln(1 + x)}{x} = 1 \]

\(|\ln(1 + a_n)| \sim |a_n|\) ,于是第一个级数也绝对收敛。

三.2.9.b

数列 \(\{a_n\}\) 满足 \(a_1 = \sqrt{\frac{1}{2}}\)\(a_{n+1} = \sqrt{\frac{1}{2} + \frac{1}{2} a_n}\) ,求 \(\prod\limits_{n=1}^{\infty} a_n\)

Answer

首先通过归纳原理不难得到 \(\forall n \in \bbN \Colon 0 < a_n < 1\) 。那么不妨令 \(a_n = \cos \theta_n\) ,其中 \(0 < \theta_n < \dfrac{\pi}{2}\)

那么首先有 \(\cos \theta_1 = \sqrt{\frac{1}{2}} = \dfrac{\pi}{4}\)

然后注意到 \(a_n = 2 a_{n+1}^2 - 1\) 而同时有二倍角公式 \(\cos 2 \alpha = 2 \cos^2 \alpha - 1\) ,于是可以知道 \(\theta_n = 2 \theta_{n+1}\) 。所以

\[ \Pi_n = \prod_{k=1}^n \cos \theta_k = \prod_{k=1}^n \cos \frac{\pi}{2^{k+1}} = \prod_{k=1}^n \frac{\sin \frac{\pi}{2^k}}{2 \sin \frac{\pi}{2^{k+1}}} = \frac{\sin \frac{\pi}{2}}{2^n \sin \frac{\pi}{2^{n+1}}} \]

而根据复合函数定理,从 \(\Lim{n \to \infty} \dfrac{\pi}{2^{n+1}} = 0\) 可以知道

\[ \lim_{n \to \infty} 2^n \sin \frac{\pi}{2^{n+1}} = \lim_{n \to \infty} \frac{\pi}{2} \frac{\sin \frac{\pi}{2^{n+1}}}{\frac{\pi}{2^{n+1}}} = \lim_{x \to 0} \frac{\pi}{2} \frac{\sin x}{x} = \frac{\pi}{2} \]

所以

\[ \prod_{n=1}^{\infty} a_n = \lim_{n \to \infty} \Pi_n = \lim_{n \to \infty} \frac{\sin \frac{\pi}{2}}{2^n \sin \frac{\pi}{2^{n+1}}} = \frac{\sin \frac{\pi}{2}}{\frac{\pi}{2}} = \frac{2}{\pi} \]